Đến nội dung

phamngochung9a nội dung

Có 451 mục bởi phamngochung9a (Tìm giới hạn từ 03-05-2020)



Sắp theo                Sắp xếp  

#670374 $P=(a^{2}+a+1)(b^{2}+b+1)(c^{2}+c+1)$.

Đã gửi bởi phamngochung9a on 29-01-2017 - 20:54 trong Bất đẳng thức - Cực trị

Cho $a,b,c\geqslant 0$ thoả mãn $a+b+c=3$. Tìm giá trị lớn nhất của biểu thức:
$P=(a^{2}+a+1)(b^{2}+b+1)(c^{2}+c+1)$.

Dạng bài tích kiểu này dùng $\text{logarit}$ hóa là hay nhất.

 

Ta có: $\ln P=\ln\left ( a^{2}+a+1 \right )+\ln \left ( b^{2}+b+1 \right )+\ln \left ( c^{2}+c+1 \right )$

 

Bằng phương pháp tiếp tuyến, ta sẽ chứng minh: $\ln (a^{2}+a+1)\leq a-1+\ln 3$   $(*)$

 

Thật vậy, xét hàm số: $f\left ( a \right )=a-1+\ln 3-\ln (a^{2}+a+1)$ trên $\left ( 0;3 \right )$ có:

 

$f'(a)=1-\frac{2a+1}{a^{2}+a+1}=\frac{a^{2}-a}{a^{2}+a+1}=0\\\Leftrightarrow a=1$

 

Vì $f'(a)$ đổi dấu từ âm sang dương khi qua điểm $a=1$ nên $f(a)$ đạt giá trị nhỏ nhất tại $a=1$

 

$\Rightarrow f(a)\geq f(1)=0$ $\Rightarrow (*)$ đúng

 

Xây dựng các bất đẳng thức tương tự rồi cộng lại, ta được:

 

$\ln P\leq a+b+c-3+3\ln 3=3\ln 3\Rightarrow P\leq 27$

 

Vậy $\max P=27$ khi và chỉ khi $a=b=c=1$




#670237 Giải hệ: $\left\{\begin{matrix} y^{2...

Đã gửi bởi phamngochung9a on 28-01-2017 - 17:00 trong Phương trình - hệ phương trình - bất phương trình

Giải hệ phương trình:

 

$1) \left\{\begin{matrix} y^{2}-2\dfrac{y}{x}+\dfrac{4}{y^{2}}=3 & \\ y^{2}x -2xy+2=2y-x& \end{matrix}\right.\left ( x,y\in \mathbb{R} \right )$

 

$2)\left\{\begin{matrix} \sqrt{-2x^{2}+13x-21}+\dfrac{2x^{2}-13x+17}{2\sqrt{2}x^{2}-13\sqrt{2}x+19\sqrt{2}}=\dfrac{\left ( y+3 \right )\sqrt{y+1}}{6\sqrt{2}} & \\ \\ \left ( x-1 \right )^{y+1}-\left ( y+1 \right )^{x-1}=0 & \end{matrix}\right.$




#670235 Chứng minh rằng: $T_{n}-\textrm{C}_{n...

Đã gửi bởi phamngochung9a on 28-01-2017 - 16:55 trong Tổ hợp - Xác suất và thống kê - Số phức

Với mỗi số thực dương $n$, đặt $T_{n}=1+\frac{1}{2}+\frac{1}{3}+...+\frac{1}{n}$. Chứng minh rằng:

 

$$T_{n}-\textrm{C}_{n}^{1}T_{n-1}+\textrm{C}_{n}^{2}T_{n-2}-...+\left ( -1 \right )^{n-1}\textrm{C}_{n}^{n-1}T_{1}=\frac{\left ( -1 \right )^{n-1}}{n}$$




#670232 CMR: $\frac{18a^3+20abc}{b+c}\ge 4b^2+4c^2...

Đã gửi bởi phamngochung9a on 28-01-2017 - 16:36 trong Bất đẳng thức - Cực trị

Cho $a,b,c$ là các số thực thỏa mãn: $\left\{\begin{matrix} a=max(a,b,c)\\b^2(17a+10c)+c^2(17a+10b)=27a^2(b+c) \end{matrix}\right.$.
Chứng minh rằng: $18a^3+20abc\ge (4b^2+4c^2+11bc)(b+c)$

Lời giải với điều kiện $a,b,c$ thực:

 

Đặt $\left\{\begin{matrix} b=xa & \\ c=ya & \end{matrix}\right.$

 

Khi đó giả thiết trở thành: $x^{2}\left ( 17+10y \right )+y^{2}\left ( 17+10x \right )=27\left ( x+y \right )$

 

Cần chứng minh: $18+20xy\geq \left ( 4x^{2}+4y^{2}+11xy \right )\left ( x+y \right )$

 

Lại đặt $\left\{\begin{matrix} x+y=S & \\ xy=P & \end{matrix}\right.$, ta có: $17\left ( S^{2}-2P \right )+10SP=27S\Rightarrow P=\frac{17S^{2}-27S}{34-10S}$

 

Khi đó: $VT-VP=18+20P-S\left ( 4S^{2}+3P \right )=18-4S^{3}+P\left ( 20-3S \right )\\=18-4S^{3}+\left ( 20-3S \right ).\frac{17S^{2}-27S}{34-10S}=\frac{\left ( S-2 \right )^{2}\left ( 40S^{2}-27S+153 \right )}{20-3S}$

 

  • Nếu $a\geq 0$ thì: $a.S=b+c\leq 2a\Rightarrow S\leq 2\Rightarrow 20-3S> 0\Rightarrow VT\geq VP$
  • Nếu $a<0$ thì: $b< 0$ và $c< 0$$\Rightarrow P> 0\Rightarrow \frac{4S^{3}-18}{20-3S}>0\Rightarrow \sqrt[3]{\frac{18}{4}}< S< \frac{20}{3}\Rightarrow 20-3S>0$

$\Rightarrow VT\geq VP$

 

Ta có điều phải chứng minh$\blacksquare$




#670052 CMR:$\frac{1}{\frac{1}{a}+...

Đã gửi bởi phamngochung9a on 26-01-2017 - 22:03 trong Bất đẳng thức và cực trị

Chứng minh rằng: Với mọi số nguyên dương $a,b,c,d$ ta có bất đẳng thức: 

$\frac{1}{\frac{1}{a}+\frac{1}{b}}+\frac{1}{\frac{1}{c}+\frac{1}{d}}\le \frac{1}{\frac{1}{a+c}+\frac{1}{b+d}}$

Bất đẳng thức tương đương với:

 

$\frac{ab}{a+b}+\frac{cd}{c+d}\leq \frac{\left ( a+c \right )\left ( b+d \right )}{a+b+c+d}\\\Leftrightarrow ab+cd+ab.\frac{c+d}{a+b}+cd.\frac{a+b}{c+d}\leq ab+bc+cd+da\\\Leftrightarrow ad\left ( \frac{b}{a+b}+\frac{c}{c+d} \right )+bc\left ( \frac{a}{a+b}+\frac{d}{c+d} \right )\leq ad+bc$

 

Ta sẽ chứng minh hai bộ số sau đơn điệu ngược chiều: $\left \{ ad,bc \right \};\left \{ \left ( \frac{b}{a+b}+\frac{c}{c+d} \right ),\left ( \frac{a}{a+b}+\frac{d}{c+d} \right ) \right \}$                 $(*)$

 

Thật vậy, xét tích: $\left ( ad-bc \right )\left ( \frac{b}{a+b}+\frac{c}{c+d}-\frac{a}{a+b}-\frac{d}{c+d} \right )=\frac{-2\left ( ac-bd \right )^{2}}{\left ( a+b \right )\left ( c+d \right )}\leq 0$

 

$\Rightarrow$  $(*)$ đúng.

 

Áp dụng bất đẳng thức $\textrm{Chebyshev}$ cho hai bộ số đơn điệu ngược chiều $\left \{ ad,bc \right \};\left \{ \left ( \frac{b}{a+b}+\frac{c}{c+d} \right ),\left ( \frac{a}{a+b}+\frac{d}{c+d} \right ) \right \}$, ta có:

 

$ad\left ( \frac{b}{a+b}+\frac{c}{c+d} \right )+bc\left ( \frac{a}{a+b}+\frac{d}{c+d} \right )\leq \frac{1}{2}\left ( ad+bc \right )\left ( \frac{b}{a+b}+\frac{c}{c+d}+\frac{a}{a+b}+\frac{d}{c+d} \right )=ad+bc$

 

$\rightarrow Q.E.D$




#669892 Tìm max của $\sum\frac{1+\sqrt{a^{2}-...

Đã gửi bởi phamngochung9a on 25-01-2017 - 17:29 trong Bất đẳng thức và cực trị

ta chứng minh $P \leq \frac{3+3\sqrt{3}}{2}$ có $P=\sum \frac{1}{a}+\sum \sqrt{1-\frac{1}{a^{2}}}\leq \sum \frac{1}{a}+\sqrt{3(3-(\sum \frac{1}{a^{2}}))}\leq \sum \frac{1}{a}+\sum \sqrt{9-(\sum \frac{1}{a})^{2}}$
đặt $\sum \frac{1}{a}=t$ ta chứng minh $t+\sqrt{9-t^{2}} \leq \frac{3+3\sqrt{3}}{2}$ $(1)$
từ đề bài chia $abc$ hai về có$1=\sum \frac{1}{ab}+\frac{2}{abc}\leq \frac{t^{2}}{3}+\frac{2t^{3}}{27}\Leftrightarrow t\geq \frac{3}{2}$

áp dụng điều vừa rồi biến đổi tương đương ở  $(1)$ thì chắc là okie :))

Cho $t=1,6$ thì $t+\sqrt{9-t^{2}}> \frac{3+3\sqrt{3}}{2}$ nhé bạn. Lần sau bạn chú ý kiểm tra thật kĩ lời giải trước khi đăng bài nhé, đừng làm giữa chừng rồi đoán bừa cho xong  :)

 

Cho các số thực a, b, c thoả mãn: $a,b,c\geqslant 1$ và $a + b + c + 2 = abc$. Tìm giá trị lớn nhất của biểu thức
$P= \frac{1+\sqrt{a^{2}-1}}{a} + \frac{1+\sqrt{b^{2}-1}}{b} +\frac{1+\sqrt{c^{2}-1}}{c}$

 

Từ đề bài, dễ thấy tồn tại các số thực dương $x,y,z$ sao cho $\left\{\begin{matrix} a=\frac{y+z}{x} & & \\ b=\frac{z+x}{y} & & \\ c=\frac{x+y}{z} & & \end{matrix}\right.$

 

Khi đó: $P=\sum \frac{x+\sqrt{\left ( y+z \right )^{2}-x^{2}}}{y+z}=\sum \frac{x+y+z+\sqrt{\left ( x+y+z \right )\left ( y+z-x \right )}}{y+z}-3\\=2\sum \frac{1+\sqrt{\frac{y+z-x}{x+y+z}}}{\frac{y+z-x}{x+y+z}+1}-3$

 

Đổi biến: $\left ( \sqrt{\frac{y+z-x}{x+y+z}},\sqrt{\frac{z+x-y}{x+y+z}},\sqrt{\frac{x+y-z}{x+y+z}} \right )\rightarrow \left ( x,y,z \right )$, khi đó:

Giả thiết trở thành $x^{2}+y^{2}+z^{2}=1$

 

Cần tìm GTLN của: $P=2\left ( \frac{1+x}{1+x^{2}}+\frac{1+y}{1+y^{2}}+\frac{1+z}{1+z^{2}} \right )-3$

 

Bằng phương pháp tiếp tuyến, ta sẽ chứng minh: $\frac{1+x}{1+x^{2}}\leq \frac{3\sqrt{3}-9}{16}.x^{2}+\frac{15+3\sqrt{3}}{16}$     $(*)$

 

Thật vậy: $(*)\Leftrightarrow \left ( 3\sqrt{3}-9 \right )\left ( x^{2}-\frac{1}{3} \right )^{2}+8\sqrt{3}\left ( x-\frac{1}{\sqrt{3}} \right )^{2}\geq 0\\\Leftrightarrow \left ( x-\frac{1}{\sqrt{3}} \right )^{2}\left [ \left ( 3\sqrt{3}-9 \right )\left ( x+\frac{1}{\sqrt{3}} \right )^{2}+8\sqrt{3} \right ]\geq 0$

 

Do $x<1$ nên $\left ( 3\sqrt{3}-9 \right )\left ( x+\frac{1}{\sqrt{3}} \right )^{2}+8\sqrt{3}> \left ( 3\sqrt{3}-9 \right )\left ( 1+\frac{1}{\sqrt{3}} \right )^{2}+8\sqrt{3}> 0$

 

$\Rightarrow (*)$ đúng.

 

Do đó: $P\leq 2\left [ \frac{3\sqrt{3}-9}{16}\left ( x^{2}+y^{2}+z^{2} \right )+\frac{9\sqrt{3}+45}{16} \right ]-3=\frac{3+3\sqrt{3}}{2}$

 

Vậy $\max P=\frac{3+3\sqrt{3}}{2}$ $\Leftrightarrow a=b=c=2$




#669891 KẾT QUẢ KỲ THI VMO 2017

Đã gửi bởi phamngochung9a on 25-01-2017 - 16:54 trong Tin tức - Vấn đề - Sự kiện

Xin được phép nêu tên một số bạn trên VMF mà mình biết là đoạt giải trong kỳ thi năm nay :)

- Lê Phước Định (phuocdinh1999)

- Nguyễn Hoàng Huy (JUV)

- Nguyễn Đức Bảo (baopbc)

- Nguyễn Cảnh Hoàng (canhhoang30011999)

- Nguyễn Đình Hoàng (Nguyen Dinh Hoang)

- Nguyễn Tiến Long (babystudymaths)

- Huỳnh Bách Khoa (dogsteven)

- Nguyễn Thành Phát (nhungvienkimcuong)

- Đỗ Hoàng Việt (vietdohoangtk7nqd)

- Nguyễn Quang Trung (ecchi123)

- Đỗ Trung Phương (vietnaminmyheart)

- Trương Mạnh Tuấn (manhtuan00)

- Phạm Ngoc Khánh (ngockhanh99k48)

Nếu có sai sót gì mong các bạn thông cảm.

Còn ai chưa có tên xin comment :D

Theo như mình biết thì còn bạn Vương Đình Ân ( vda2000 ) được giải nhì nữa  :lol:




#669252 Max $P=\frac{\sqrt{xyz}}{(1+x)(1+x+y)...

Đã gửi bởi phamngochung9a on 21-01-2017 - 22:24 trong Bất đẳng thức và cực trị

Cho $x;y;z$ là các số dương tùy ý . Tìm giá trị lớn nhất của biểu thức $P=\frac{\sqrt{xyz}}{(1+x)(1+x+y)(1+x+y+z)}$

Xét mẫu của $P$:

$A=\left ( 1+x \right )\left ( 1+x+y \right )\left ( 1+x+y+z \right )$

$\geq 2\left ( 1+x \right )\left ( 1+x+y \right )\sqrt{z\left ( 1+x+y \right )}$

$\Rightarrow A^{2}\geq 4\left ( 1+x \right )^{2}\left ( 1+x+y \right )^{3}z=4\left ( 1+x \right )^{2}\left ( \frac{1+x}{2}+\frac{1+x}{2}+y \right )^{3}z$

$\geq 108\left ( 1+x \right )^{2}.\frac{\left ( 1+x \right )^{2}}{4}.yz=27\left ( \frac{1}{3}+\frac{1}{3}+\frac{1}{3}+x \right )^{4}yz$

$\geq 256xyz$

$\Rightarrow P\leq \frac{\sqrt{xyz}}{A}\leq \frac{1}{\sqrt{256}}=\frac{1}{16}$

 

Vậy $\max P = \frac{1}{16}$ khi và chỉ khi: $\left\{\begin{matrix} x=\frac{1}{3} & & \\ y=\frac{2}{3} & & \\ z=2 & & \end{matrix}\right.$




#669240 CMR: $\sqrt{a+b}+\sqrt{b+c}+\sqrt...

Đã gửi bởi phamngochung9a on 21-01-2017 - 21:47 trong Bất đẳng thức - Cực trị

Cho $a,b,c$ là các số thực không âm thỏa mãn: $a^2+b^2+c^2=1$. 

Chứng minh rằng:

$\sqrt{a+b}+\sqrt{b+c}+\sqrt{c+a}\geq \sqrt{7(a+b+c)-3}$

Bình phương hai vế, ta có:

CodeCogsEqn.gif

Mà:

CodeCogsEqn (1).gif

Đặt $a+b+c=t$, ta cần chứng minh:

CodeCogsEqn (2).gif

 

Do $\left\{\begin{matrix} a+b+c\geq \sqrt{a^{2}+b^{2}+c^{2}}=1 & \\ a+b+c\leq \sqrt{3\left (a^{2}+b^{2}+c^{2} \right )}=\sqrt{3} & \end{matrix}\right.\Rightarrow t\in \left [ 1;\sqrt{3} \right ]$

 

Nên $(*)$ luôn đúng. Ta có điều phải chứng minh.




#664898 tìm gtnn

Đã gửi bởi phamngochung9a on 17-12-2016 - 19:19 trong Đại số

Pn ơi cho mik hỏi xíu nhé! cái chỗ đầu tiên ấy là dựa vào BĐT nào z hay biến đổi ntn z?

  • Trên tử đánh giá dựa vào BĐT Holder : $\left ( x^{3}+y^{3}+z^{3} \right )\left ( 1+1+1 \right )\left ( 1+1+1 \right )\geq \left ( x+y+z \right )^{3}\\\Rightarrow x^{3}+y^{3}+z^{3}\geq \frac{\left ( x+y+z \right )^{3}}{9}$
  • Dưới mẫu đánh giá qua BĐT phụ quen thuộc: $ab+bc+ca\leq \frac{\left ( a+b+c \right )^{2}}{3}$



#664867 chứng minh BĐT

Đã gửi bởi phamngochung9a on 17-12-2016 - 00:08 trong Bất đẳng thức và cực trị

Chỗ dấu bé hơn hoặc bằng đầu tiên ấy bạn, bạn có thể lí giải kĩ hơn được không?

Là BĐT Buniakovsky nhé bạn:

$\sum \sqrt{\left ( xy+xz \right )\left ( x+z \right )}\\= \sqrt{\left ( xy+xz \right )\left ( x+z \right )}+\sqrt{\left ( yx+yz \right )\left ( x+y \right )}+\sqrt{\left ( zx+zy \right )\left ( y+z \right )}\\\leq \sqrt{\left ( 2xy+2yz+2zx \right )\left ( 2x+2y+2z \right )}\\=2\sqrt{\left ( xy+yz+zx \right )\left ( x+y+z \right )}$




#664862 chứng minh BĐT

Đã gửi bởi phamngochung9a on 16-12-2016 - 23:38 trong Bất đẳng thức và cực trị

Cho abc=(1-a)(1-b)(1-c) và a;b;c $\epsilon (0;1)$

CMR :$\sqrt{a} +\sqrt{b} +\sqrt{c} \leq \frac{3\sqrt{2}}{2}$

Từ đề bài, ta có: $\left ( \frac{1}{a}-1 \right )\left ( \frac{1}{b}-1 \right )\left ( \frac{1}{c}-1 \right )=1$

 

Đặt $\left\{\begin{matrix} \frac{1}{a}-1=\frac{y}{x} & & \\ \frac{1}{b}-1=\frac{z}{y} & & \\ \frac{1}{c}-1=\frac{x}{z} & & \end{matrix}\right.$.

Khi đó: $VT=\sqrt{\frac{x}{x+y}}+\sqrt{\frac{y}{y+z}}+\sqrt{\frac{z}{z+x}}\\=\frac{\sum \sqrt{\left ( xy+xz \right )\left ( z+x \right )}}{\sqrt{\left ( x+y \right )\left ( y+z \right )\left ( z+x \right )}}\leq \frac{\sqrt{2\left ( xy+yz+zx \right )}.\sqrt{2\left ( x+y+z \right )}}{\sqrt{\left ( x+y \right )\left ( y+z \right )\left ( z+x \right )}}\\=\frac{2\sqrt{\left ( xy+yz+zx \right )\left ( x+y+z \right )}}{\sqrt{\left ( x+y \right )\left ( y+z \right )\left ( z+x \right )}}\\$

 

Ta sẽ chứng minh: $\left ( x+y \right )\left ( y+z \right )\left ( z+x \right )\geq \frac{8}{9}\left ( x+y+z \right )\left ( xy+yz+zx \right )$. 

Thật vậy, BĐT trên tương đương với: $9\left ( x+y \right )\left ( y+z \right )\left ( z+x \right )\geq 8\left ( x+y+z \right )\left ( xy+yz+zx \right )\\\Leftrightarrow \left ( x+y+z \right )\left ( xy+yz+zx \right )\geq 9xyz$  (luôn đúng theo $AM-GM$)

 

Vậy: $VT\leq \frac{2\sqrt{\left ( x+y+z \right )\left ( xy+yz+zx \right )}}{\sqrt{\frac{8}{9}\left ( x+y+z \right )\left ( xy+yz+zx \right )}}=\frac{3\sqrt{2}}{2}$

 

Dấu $"="$ xảy ra $x=y=z\Leftrightarrow a=b=c=\frac{\sqrt{2}}{2}$




#664859 tìm gtnn

Đã gửi bởi phamngochung9a on 16-12-2016 - 23:05 trong Đại số

Cho x,y,z là các số thực dương.Tìm giá trị nhỏ nhất của biểu thức:
P=$\fn_jvn \frac{3(x^3+y^3+z^3)}{4(xy+yz+xz)}+\frac{1}{(x+y+z)^2}$

Có phải ý bạn là:

 

 

 

Cho x,y,z là các số thực dương.Tìm giá trị nhỏ nhất của biểu thức:
P=$\frac{3(x^3+y^3+z^3)}{4(xy+yz+xz)}+\frac{1}{(x+y+z)^2}$

 

Ta có: $P\geq \frac{\frac{3}{9}\left ( x+y+z \right )^{3}}{\frac{4}{3}\left ( x+y+z \right )^{2}}+\frac{1}{\left ( x+y+z \right )^{2}}\\= \frac{x+y+z}{4}+\frac{1}{\left ( x+y+z \right )^{2}}\\\geq 3\sqrt[3]{\frac{1}{64}}=\frac{3}{4}$

 

Vậy $\min P=\frac{3}{4}\Leftrightarrow a=b=c=\frac{2}{3}$




#664855 Cho ba số a,b,c không đồng thời bằng 0 thỏa mãn $ a^{2}+b^...

Đã gửi bởi phamngochung9a on 16-12-2016 - 22:51 trong Bất đẳng thức và cực trị

Cho ba số a,b,c không đồng thời bằng 0 thỏa mãn $ a^{2}+b^{2}+c^{2} \leq 3 $. Tìm GTLN của biểu thức:

$ P=ab-bc-ca-\frac{2016}{a^{2}+b^{2}+c^{2}} $

Ta có: $ab-bc-ca\leq \left | ab \right |+\left | bc \right |+\left | ca \right |\leq a^{2}+b^{2}+c^{2}\leq 3$

 

Vậy: $P\leq 3-\frac{2016}{3}=-669$

 

$\Rightarrow \max P=-669\Leftrightarrow \left\{\begin{matrix} a=b=1 & \\ c=-1 & \end{matrix}\right.$




#664306 cho x,y,z là các số thực dương

Đã gửi bởi phamngochung9a on 10-12-2016 - 19:43 trong Bất đẳng thức và cực trị

cho x,y,z là các số thực dương thỏa mãn $\frac{2}{3x+2y+z+1}+\frac{2}{3x+y+2z+1}= (x+y)(x+z)$.Tìm max của P=$\frac{2(x+3)^{2}+y^{2}+z^{2}-16}{2x^{2}+y^{2}+z^{2}}$

Lâu rồi mới thấy anh Bí đăng bài  :D

 

Theo BĐT Cauchy-Schwarz, ta có:

 

$\left ( x+y \right )\left ( x+z \right )\geq \frac{8}{6x+3y+3z+2}$

 

Đặt $\left\{\begin{matrix} x+y=a & \\ x+z=b & \end{matrix}\right.$, ta có:

 

$ab\geq \frac{8}{3a+3b+2}\Rightarrow \frac{\left ( a+b \right )^{2}}{4}\geq \frac{8}{3\left ( a+b \right )+2}\Rightarrow a+b\geq 2$

 

$\Rightarrow y+z\geq 2-2x$

  • Nếu $x\geq 1$ thì: 

$P=\frac{12x+2}{2x^{2}+y^{2}+z^{2}}+1< \frac{1+6x}{x^{2}}+1=\frac{1}{x^{2}}+\frac{6}{x}+1\leq 8$

  • Nếu $x<1$ thì: 

Ta có: $\left ( y+z \right )^{2}\geq 4\left ( x-1 \right )^{2}$

 

$P=\frac{12x+2}{2x^{2}+y^{2}+z^{2}}+1\leq \frac{12x+2}{2x^{2}+\frac{\left ( y+z \right )^{2}}{2}}+1\leq \frac{12x+2}{2x^{2}+\frac{\left ( 2-2x \right )^{2}}{2}}+1=\frac{6x+1}{2x^{2}-2x+1}+1$

 

Xét hàm số $f\left ( t \right )=\frac{6t+1}{2t^{2}-2t+1}$ trên $\left ( 0;1 \right )$ có:

 

$f'(t)=\frac{-12t^{2}-4t+8}{\left ( 2t^{2}-2t+1 \right )^{2}}=0\Leftrightarrow t=\frac{2}{3}$

 

$\Rightarrow P\leq f\left ( \frac{2}{3} \right )+1=10$

 

Vậy $\max P=10\Leftrightarrow \left\{\begin{matrix} x=\frac{2}{3} & & \\ y=\frac{1}{3} & & \\ z=\frac{1}{3} & & \end{matrix}\right.$




#662216 Tính xác suất để chọn được 2 trong số 80 em không học cả Anh văn lẫn Pháp văn

Đã gửi bởi phamngochung9a on 17-11-2016 - 12:31 trong Tổ hợp - Xác suất và thống kê - Số phức

Kết quả chưa đúng bạn ơi.

Mình giải sai ở đâu vậy bạn  :(




#662056 Tính xác suất để chọn được 2 trong số 80 em không học cả Anh văn lẫn Pháp văn

Đã gửi bởi phamngochung9a on 15-11-2016 - 19:49 trong Tổ hợp - Xác suất và thống kê - Số phức

Bài toán tính xác suất

  • Số học sinh chỉ học Anh văn là: $40-20=20$ (em)
  • Số học sinh chỉ học Pháp văn là: $30-20=10$ (em)
  • Số học sinh không học cả Anh văn lẫn Pháp văn là: $80-20-10-20=30$ (em)

Vậy xác suất để chọn được $2$ em không học cả Anh văn lẫn Pháp văn là: $\frac{C_{30}^{2}}{C_{80}^{2}}=\frac{87}{632}$




#661379 Chứng minh $\frac{x^2}{yz}+\frac{4...

Đã gửi bởi phamngochung9a on 10-11-2016 - 19:49 trong Bất đẳng thức và cực trị

Cho 3 số thực dương x, y, z thỏa mãn: 3y2z2+x2=2(x+yz).

CMR:

$\frac{x^2}{yz}+\frac{4}{\left(x+y\right)^2}+\frac{4}{\left(x+z\right)^2}\ge 3$

Bằng biến đổi tương đương, dễ dàng chứng minh bất đẳng thức:

 

$$\frac{4}{\left ( x+y \right )^{2}}+\frac{4}{\left ( x+z \right )^{2}}\geq \frac{4}{x^{2}+yz}$$

 

$$\Rightarrow VT\geq \frac{x^{2}}{yz}+\frac{4}{x^{2}+yz}$$

 

Từ đề bài, ta có: 

 

$3y^{2}z^{2}+x^{2}\leq x^{2}+1+2yz\\\Rightarrow 3y^{2}z^{2}-2yz-1\leq 0\Rightarrow yz\leq 1$

 

Khi đó:

 

$VT\geq x^{2}+\frac{4}{x^{2}+1}=\left ( x^{2}+1 \right )+\frac{4}{x^{2}+1}-1\geq 3$

 

Dấu $"="$ xảy ra khi và chỉ khi $x=y=z=1$




#656690 Đề thi chọn đội tuyển Quốc gia tỉnh Vũng Tàu năm 2016-2017

Đã gửi bởi phamngochung9a on 04-10-2016 - 19:50 trong Thi HSG cấp Tỉnh, Thành phố. Olympic 30-4. Đề thi và kiểm tra đội tuyển các cấp.

Cách khác cho Bài 1. 2) 

 

Ta có:

 

$x+y+z=\left ( x^{2}+y^{2}+z^{2} \right )\left ( x+y+z \right )\geq 3\left ( x^{2}y+y^{2}z+z^{2}x \right )$

 

Do đó:

$VT\leq \frac{x+y+z}{3}\left ( \sum \frac{1}{\sqrt{1+x^{2}}} \right )$

 

Bằng phương pháp tiếp tuyến dễ dàng chỉ ra $\frac{1}{\sqrt{1+a^{2}}}\leq -\frac{3}{8}a+\frac{5\sqrt{3}}{8}$    $\forall a\in \left [ 0;1 \right ]$

 

Vậy:

$VT\leq \frac{x+y+z}{3}.\left [-\frac{3}{8}\left ( x+y+z \right )+\frac{15\sqrt{3}}{8} \right ]\\=-\frac{1}{8}\left ( x+y+z \right )^{2}+\frac{5\sqrt{3}}{8}\left ( x+y+z \right )\\=-\frac{1}{8}\left ( x+y+z-4\sqrt{3} \right )\left ( x+y+z-\sqrt{3} \right )+\frac{3}{2}$

 

Vì $x+y+z\leq \sqrt{3}$ nên

 

$VT\leq \frac{3}{2}=VP\rightarrow Q.E.D$

 

Đẳng thức xảy ra $\Leftrightarrow x=y=z=\frac{1}{\sqrt{3}}$




#656131 Đề thi chọn đội tuyển Quốc gia tỉnh Thanh Hóa năm 2016-2017 ngày 1

Đã gửi bởi phamngochung9a on 30-09-2016 - 19:40 trong Thi HSG cấp Tỉnh, Thành phố. Olympic 30-4. Đề thi và kiểm tra đội tuyển các cấp.

De hsg Thanh Hoa.jpg




#656015 Tìm GTNN của $P=a+b+c+\frac{1}{abc}+\frac...

Đã gửi bởi phamngochung9a on 29-09-2016 - 20:03 trong Bất đẳng thức - Cực trị

Không mất tính tổng quát giả sử $a \geq b \geq c $

Từ đầu bài suy ra được là $\sqrt{b} + \sqrt{c} = \sqrt{a} $

Thay vô lại còn bđt 2 biến thôi

Rồi sau đó S,P là ra

Ý tưởng của bạn rất hay nhưng bạn lại không giải bài toán cho rõ ràng, mình mong bạn nên xem xét lại điều này. Nếu bạn còn vi phạm nữa thì buộc mình phải nhắc nhở bạn lỗi spam thôi. Hãy cùng nhau xây dựng một $VMF$ phát triển nhé  :D  :lol:

 

Cho $a,b,c>0$ thỏa mãn $a^2+b^2+c^2=2ab+2bc+2ca$. Tìm GTNN của biểu thức

$$P=a+b+c+\frac{1}{abc}+\frac{9}{a+b+c}$$

 

Một lời giải theo hướng khác của mình:

 

Biến đổi giả thiết trở thành: $\left ( a+b+c \right )^{2}=4\left ( ab+bc+ca \right )$

 

Đặt $\left\{\begin{matrix} \frac{a}{a+b+c}=x & & \\ \frac{b}{a+b+c}=y & & \\ \frac{c}{a+b+c}=z & & \end{matrix}\right.$, ta có:

 

$$\left\{\begin{matrix} x+y+z=1 & \\ xy+yz+zx=\frac{1}{4} & \end{matrix}\right.$$

 

Ta sẽ tìm $GTLN$ của $xyz$. Ta có:

 

$xyz=z\left [ \frac{1}{4}-z\left ( x+y \right ) \right ]=z\left ( z^{2}-z+\frac{1}{4} \right )$

 

Mặt khác: $\frac{1}{4}=xy+yz+zx\leq \frac{\left ( x+y \right )^{2}}{4}+z\left ( 1-z \right )=-\frac{3}{4}z^{2}+\frac{1}{2}z+\frac{1}{4}\\\Rightarrow 3z^{2}-2z\leq 0\Rightarrow z\in \left [0;\frac{2}{3} \right ]$ 

 

Xét hàm số $f(z)=z\left ( z^{2}-z+\frac{1}{4} \right )$ trên $\left [0;\frac{2}{3} \right ]$, ta có: $f\left ( z \right )\leq \frac{1}{54}$

$\Rightarrow xyz\leq \frac{1}{54}$

$\Rightarrow \frac{abc}{\left ( a+b+c \right )^{3}}\leq \frac{1}{54}\\\Rightarrow \frac{1}{abc}\geq \frac{54}{\left ( a+b+c \right )^{3}}$

 
Khi đó: $P\geq a+b+c+\frac{54}{\left ( a+b+c \right )^{3}}+\frac{9}{a+b+c}$
 
Đặt $a+b+c=t$, ta có: $P\geq t+\frac{9}{t}+\frac{54}{t^{3}}$
 
Khảo sát hàm $f(t)$ trên $(0;+\infty)$, ta được $P\geq 5\sqrt{2}$
 
Vậy $\min P=5\sqrt{2}\Leftrightarrow \left\{\begin{matrix} a=\frac{\sqrt{2}}{2} & & \\ b=\frac{\sqrt{2}}{2} & & \\ c=2\sqrt{2} & & \end{matrix}\right.$ và các hoán vị



#653894 MIN: $P=\frac{1}{(a+1)^2}+\frac{1...

Đã gửi bởi phamngochung9a on 12-09-2016 - 17:56 trong Bất đẳng thức và cực trị

Cho $a,b,c> 0$ và $abc=1$. Tìm GTNN của:

$P=\frac{1}{(a+1)^2}+\frac{1}{(b+1)^2}+\frac{1}{(c+1)^2}+\frac{1}{a+b+c+1}$

Theo nguyên lí $Dirichlet$, giả sử $\left ( a-1 \right )\left ( b-1 \right )\geq 0$, ta có: 

$$\left ( a-1 \right )\left ( b-1 \right )\geq 0\Rightarrow a+b\leq ab+1$$

 

Khi đó:

 

$$P=\sum \frac{1}{\left ( a+1 \right )^{2}}+\frac{1}{a+b+c+1}\geq \frac{1}{ab+1}+\frac{1}{\left ( c+1 \right )^{2}}+\frac{1}{a+b+c+1}\geq \frac{1}{\frac{1}{c}+1}+\frac{1}{\left ( c+1 \right )^{2}}+\frac{1}{\frac{1}{c}+c+2}=\frac{c}{c+1}+\frac{1}{\left ( c+1 \right )^{2}}+\frac{c}{\left ( c+1 \right )^{2}}=1$$

 

Vậy $\min P=1\Leftrightarrow a=b=c=1$




#653353 Thăm dò ý kiến về việc thi trắc nghiệm môn toán

Đã gửi bởi phamngochung9a on 08-09-2016 - 19:10 trong Tin tức - Vấn đề - Sự kiện

Đúng vậy, môn khác trắc nghiệm còn có lí chứ toán mà trắc nghiệm thì đâu còn là toán nữa. Như kiểu tính tích phân, giải PT,... chỉ có cái máy tính là xong chứ mấy, đâu cần nháp nữa. Còn hình $Oxy$ thì cứ đoán bừa mấy cái quan hệ chứ đâu cần chứng minh. BĐT thì dự đoán dấu bằng là được, chẳng cần làm gì cho mệt. Thế là ba câu phân loại mất hết còn gì nữa. 

 

Mới nghe toán trắc nghiệm thôi mà ức chế hết cỡ  :angry:  :angry:




#653172 Giải pt: $cos^2\left ( x - \frac{\pi }{6...

Đã gửi bởi phamngochung9a on 07-09-2016 - 20:06 trong Phương trình, Hệ phương trình Lượng giác

Giải pt: $\cos^2\left ( x - \frac{\pi }{6} \right ) -\sin^2\left ( x -\frac{\pi }{6} \right )= \cos\left ( x + \frac{\pi }{3} \right )$

Phương trình đã cho tương đương với:

 

$$\cos^{2}\left ( x-\frac{\pi}{6} \right )-\sin^{2} \left ( x-\frac{\pi}{6} \right )=\cos\left [ \left ( x-\frac{\pi}{6} \right )+\frac{\pi}{2} \right ]\\\Leftrightarrow \cos^{2}\left ( x-\frac{\pi}{6} \right )-\sin^{2} \left ( x-\frac{\pi}{6} \right )=-\sin\left ( x-\frac{\pi}{6} \right )\\$$

 

Đặt $\sin\left ( x-\frac{\pi}{6} \right )=t$, ta có:

$$1-t^{2}-t^{2}=-t \\\Leftrightarrow 2t^{2}-t-1=0\\\Leftrightarrow \begin{bmatrix} t=1 & \\ t=-\frac{1}{2} & \end{bmatrix}$$

 

  • Với $t=1$, ta có:

$$\sin\left ( x-\frac{\pi}{6} \right )=1\\\Leftrightarrow x=\frac{2\pi}{3}+k2\pi$$

  • Với $t=-\frac{1}{2}$, ta có:

$$\sin\left ( x-\frac{\pi}{6} \right )=-\frac{1}{2}\\\Leftrightarrow \begin{bmatrix} x=k2\pi & \\ x=\frac{4\pi}{3}+k2\pi & \end{bmatrix}$$

 

Vậy.......... :icon10:




#653043 $\frac{a+1}{b^2+1}+\frac{b+1}...

Đã gửi bởi phamngochung9a on 06-09-2016 - 20:30 trong Bất đẳng thức và cực trị

Cho $a,b,c\in \left [ 0;1 \right ] a+b+c=2$

Tìm min: P= $\frac{a+1}{b^2+1}+\frac{b+1}{c^2+1}+\frac{c+1}{a^2+1}+\frac{3}{4}(ab+bc+ca)$

Bằng $UTC$, ta sẽ chỉ ra rằng: 

 

$$\frac{1}{x^{2}+1}\geq 1-\frac{x}{2} \quad \quad \forall x \in \left [ 0;1 \right ]$$

 

Thật vậy, BĐT trên tương đương với: $x^{3}-2x^{2}+x\geq 0\Leftrightarrow x\left ( x-1 \right )^{2}\geq 0$    ( luôn đúng )

 

Vậy: $P\geq \sum \left ( a+1 \right )\left ( 1-\frac{b}{2} \right )+\frac{3}{4}\left ( ab+bc+ca \right )\\=4+\frac{1}{4}\left ( ab+bc+ca \right )$

 

Do đó, ta chỉ cần chứng minh: $ab+bc+ca\geq 1$

 

Ta có: $\left ( a-1 \right )\left ( b-1 \right )\left ( c-1 \right )\leq 0\\\Leftrightarrow ab+bc+ca\geq 1+abc\geq 1$

 

$\Rightarrow P\geq \frac{17}{4}$

 

Vậy $\min P=\frac{17}{4}\Leftrightarrow \left\{\begin{matrix} a=1 & & \\ b=1 & & \\ c=0 & & \end{matrix}\right.$ và các hoán vị 

 

Bài toán được giải quyết $\blacksquare$